Domanda:
In questa griglia infinita di resistori, qual è la resistenza equivalente?
Malabarba
2010-12-20 06:11:13 UTC
view on stackexchange narkive permalink

Ho cercato e non sono riuscito a trovarlo sul sito, quindi eccolo qui (citato alla lettera):

Su questa griglia infinita di resistenze da un ohm ideali qual è la resistenza equivalente tra i due nodi contrassegnati?

Nerd Sniping

Con un collegamento alla fonte.

Non sono proprio sicuro che ci sia una risposta a questa domanda. Tuttavia, data la mia mancanza di esperienza con l'elettronica di base, potrebbe anche essere facile.

Ho riconosciuto immediatamente il titolo di XKCD [Nerd Snipping è uno dei miei preferiti].
Discussione sul meta: http://meta.physics.stackexchange.com/q/253/
[La domanda @ m.SE ...] (http://math.stackexchange.com/questions/12863)
@MarkEichenlaub Per quanto riguarda il tuo secondo commento: non ho il tempo o la volontà di dettagliare ogni singolo risultato, ma essenzialmente ci sono tre diversi problemi affrontati (adiacente, diagonale e "mossa del cavaliere", o anche quattro se conti la soluzione generale) . La soluzione principale è lunga oltre 2800 parole, entra in molti dettagli matematici e risolve solo il problema diagonale generale. Sento che la domanda necessita ancora di una risposta concisa, chiara e organizzata che sia facile da trovare.
È un duplicato, ma è troppo tardi per chiuderlo; Lo chiamerò un "buon duplicato" e me ne andrò in pace.
Correlati: [Il problema "Nerd Sniping". Generalizzazioni?] (Http://physics.stackexchange.com/questions/10308/)
Due risposte:
Sklivvz
2010-12-20 06:38:13 UTC
view on stackexchange narkive permalink

Nerd Sniping!

La risposta è $ \ frac {4} {\ pi} - \ frac {1} {2} $.

Spiegazione semplice:

Approssimazione successiva! Inizierò con il caso più semplice (vedi immagine sotto) e aggiungerò sempre più resistori per provare ad approssimare una griglia infinita di resistori.

Simulation results

Derivazione matematica:

$$ R_ {m, m} = \ frac 2 \ pi \ left (1 + \ frac 13 + \ frac 15 + \ frac 17 + \ dots + \ frac 1 {2m-1} \ right) $$

+1, ma sarebbe ancora meglio delineare la soluzione nel post in modo che le persone non debbano fare clic su un collegamento per vedere come è fatto.
Le cose su quel collegamento matematico sono piuttosto complicate ... Troppo per semplici forme di vita disumane come me.
Sì, mi ci sono volute alcune letture per capire come è stato fatto. (Questo è ciò che lo rende "divertente" :-P)
@David Zaslavsky: In questo caso particolare, probabilmente sono molto più soddisfatto dal calcolo numerico che dalla derivazione esatta.
-1
-1 Questa non è definitivamente una spiegazione semplice, voglio vedere semplice. Tutto ciò di cui hai bisogno per risolvere questo problema è la legge di Ohm insieme a quella di Kirchoff.
spiegazione! = simulazione. inoltre voglio conoscere la soluzione generale per due punti arbitrari.
@kalle43 Non conosco modo migliore di affrontare il problema di quelli descritti nei link di @Sklivvz. Solo perché vuoi semplice non significa che esista. Se è così facile, perché non trovi tu stesso una risposta semplice!
@Mark Ho presto una dimostrazione elementare, ma non sono così sicuro di pubblicarla qui.
@kalle43: la "simulazione" è anche una spiegazione (molto più della semplice matematica). Se consideri il circuito "interno" e aggiungi sempre più strati, vedi che il loro contributo è sempre meno significativo. Quindi in pratica il circuito funziona praticamente a livello locale e le resistenze più di, diciamo 20 passi di distanza sono insignificanti.
La legge di @kalle: Kirchhoff è quella che ho menzionato nel mio commento sopra. Otterrai una matrice a dimensione infinita e dovrai calcolarne la determinante. Oppure puoi usare varie dualità che collegano la rete di resistori con modelli in fisica statistica. Tuttavia, dubito fortemente che qualsiasi metodo possibile sia in qualche modo facile. Avrai sicuramente bisogno di fare la trasformata di Fourier o integrali non banali (come nel collegamento di Sklivvz) ad un certo punto per ottenere i risultati. Quindi stai dicendo che hai ottenuto qualcosa di semplice che può battere questi metodi consolidati? Non posso dire di non dubitare di te ;-)
@JOHA: I margini qui sono probabilmente troppo piccoli per la tua prova comunque. ;)
Viene richiesta la generalizzazione di @JOHA: [qui] (http://physics.stackexchange.com/questions/10308/the-nerd-sniping-problem-generalizations)
$ R_ {m, m} $ è la resistenza tra $ (0,0) $ e $ (m, m) $, cioè sulla diagonale.La domanda chiede circa $ R_ {2,1} $, quindi l'inclusione di $ R_ {m, m} $ nella risposta è gratuita e dovrebbe probabilmente essere eliminata.
E in qualche modo otteniamo pi greco senza cerchi.La matematica è fantastica.
preferisco l'approssimazione di 0,773 come risposta.ma, d'altra parte, quell'autobus probabilmente non mi avrebbe mai colpito, in qualche modo triste.:(
PBS
2018-04-21 20:25:44 UTC
view on stackexchange narkive permalink

Ecco la mia derivazione preferita, che è più o meno basata su, ma a mio parere piuttosto più semplice di quelle fornite nei link sopra. È necessaria solo l'integrazione elementare!

La configurazione

Lavora su una griglia $ N $ -dimensionale per la generalità ed etichetta i punti della griglia con $ \ vec {n} $, un vettore intero.

Supponiamo che la tensione in ogni punto sia $ V_ \ vec {n} $. Quindi la corrente che scorre in $ \ vec {n} $ dai suoi $ 2N $ vicini è

$$ \ sum_ {i, \ pm} (V _ {\ vec {n} \ pm \ vec {e} _i} - V_ \ vec {n}) $$

($ \ vec {e} _i $ è il vettore unitario lungo la direzione $ i $.)

Insistere sul fatto che una fonte esterna sta pompando un amplificatore in $ \ vec {0} $ e fuori da $ \ vec {a} $. L'attuale conservazione dà

$$ \ sum_ {i, \ pm} (V _ {\ vec {n} \ pm \ vec {e} _i} - V_ \ vec {n}) = - \ delta_ \ vec {n} + \ delta_ {\ vec {n} - \ vec {a}} $$

($ \ delta_ \ vec {n} $ è uguale a $ 1 $ se $ \ vec {n} = \ vec {0} $ e $ 0 $ altrimenti.)

Questa è l'equazione che vogliamo risolvere. Dato $ V_ \ vec {n} $, la resistenza effettiva tra $ \ vec {0} $ e $ \ vec {a} $ è semplicemente

$$ R_ \ vec {a} = V_ \ vec {0} - V_ \ vec {a} $$

Sfortunatamente, ci sono infinite soluzioni per $ V_ \ vec {n} $ e i loro risultati per $ R_ \ vec {a} $ non sono d'accordo! Questo perché la domanda non specifica alcuna condizione al contorno all'infinito. A seconda di come li scegliamo, possiamo ottenere qualsiasi valore di $ R_ \ vec {a} $ che ci piace! Risulterà che esiste una scelta ragionevole unica, ma per ora dimentichiamoci completamente di questo problema e troviamo solo qualsiasi soluzione.

Soluzione mediante trasformata di Fourier

La strategia è trovare una funzione verde $ G_ \ vec {n} $ soddisfacente

$$ \ sum_ {i, \ pm} (G _ {\ vec {n} \ pm \ vec {e} _i} - G_ \ vec {n}) = \ delta_ \ vec {n} $$

Una soluzione all'equazione originale sarebbe quindi

$$ V_n = -G_ \ vec {n} + G _ {\ vec {n} - \ vec {a}} $$

Per trovare $ G_ \ vec {n} $, supponiamo che possa essere rappresentato come

$$ G_ \ vec {n} = \ int_0 ^ {2 \ pi} \ frac {d ^ N \ vec {k}} {(2 \ pi) ^ N} (e ^ {i \ vec {k } \ cdot \ vec {n}} - 1) g (\ vec {k}) $$

Quindi notandolo

\ begin {align} \ sum_ {i, \ pm} (G _ {\ vec {n} \ pm \ vec {e} _i} - G_ \ vec {n}) & = \ int_0 ^ {2 \ pi} \ frac {d ^ N \ vec {k}} {(2 \ pi) ^ N} e ^ {i \ vec {k} \ cdot \ vec {n}} \ left (\ somma_ {i, \ pm} e ^ {\ pm i k_i} - 2N \ right) g (\ vec {k}) \\ \ delta_ \ vec {n} & = \ int_0 ^ {2 \ pi} \ frac {d ^ N \ vec {k}} {(2 \ pi) ^ N} e ^ {i \ vec {k} \ cdot \ vec {n}} \ end {align}

vediamo che l'equazione per $ G_ \ vec {n} $ può essere risolta scegliendo

$$ g (\ vec {k}) = \ frac {1} {\ sum_ {i, \ pm} e ^ {\ pm k_i} - 2N} $$

che porta a

$$ G_ \ vec {n} = \ frac {1} {2} \ int_0 ^ {2 \ pi} \ frac {d ^ N \ vec {k}} {(2 \ pi) ^ N} \ frac {\ cos (\ vec {k} \ cdot \ vec {n}) - 1} {\ sum \ cos (k_i) - N} $$

A proposito, il divertente $ -1 $ al numeratore non sembra fare molto altro che spostare $ G_ \ vec {n} $ di una $ \ vec {n} $ - costante indipendente, quindi potrebbe chiedersi cosa ci fa lì. Ma senza di esso l'integrale sarebbe infinito, almeno per $ N \ leq 2 $.

Quindi la risposta finale è

$$ R_ \ vec {a} = V_ \ vec {0} - V_ \ vec {a} = 2 (G_ \ vec {a} - G_ \ vec {0}) = \ int_0 ^ {2 \ pi } \ frac {d ^ N \ vec {k}} {(2 \ pi) ^ N} \ frac {1 - \ cos (\ vec {k} \ cdot \ vec {a})} {N - \ sum \ cos (k_i)} $$

Perché questa è la risposta giusta?

(Da questo punto in poi, $ N = 2 $)

Ho detto prima che c'erano infinite soluzioni per $ V_ \ vec {n} $. Ma quello sopra è speciale, perché a grandi distanze $ r $ dall'origine, le tensioni e le correnti si comportano come

$$ V = \ mathcal {O} (1 / r) \ qquad I = \ mathcal {O} (1 / r ^ 2) $$

Un teorema standard (unicità delle soluzioni dell'equazione di Laplace) dice che può esserci solo una soluzione che soddisfa questa condizione. Quindi la nostra soluzione è l'unica con il minor flusso di corrente possibile all'infinito e con $ V_ \ infty = 0 $ . E anche se la domanda non lo chiedesse, è ovviamente l'unica cosa ragionevole da chiedere.

Oppure lo è? Forse preferiresti definire il problema lavorando su una griglia finita, trovando la soluzione unica per $ V_ \ vec {n} $ lì, quindi cercando di prendere una sorta di limite mentre la dimensione della griglia va all'infinito. Tuttavia, si può sostenere che $ V_ \ vec {n} $ ottenuto da una griglia size- $ L $ dovrebbe convergere al nostro $ V_ \ vec {n} $ con un errore dell'ordine $ 1 / L $. Quindi il risultato finale è lo stesso.

Fare gli integrali

Considera innanzitutto il caso diagonale

\ begin {align} R_ {n, n} & = \ frac {1} {(2 \ pi) ^ 2} \ int_A dx \, dy \, \ frac {1 - \ cos (n (x + y))} {2 - \ cos (x) - \ cos (y)} \\ & = \ frac {1} {2 (2 \ pi) ^ 2} \ int_A dx \, dy \, \ frac {1 - \ cos (n (x + y))} {1 - \ cos (\ frac { x + y} {2}) \ cos (\ frac {xy} {2})} \ end {align}

dove $ A $ è il quadrato $ 0 \ leq x, y \ leq 2 \ pi $.

Poiché l'integrando è periodico, il dominio può essere modificato da $ A $ a $ A '$ in questo modo:

Rectangles A and A'

Quindi modificare le variabili in

$$ a = \ frac {x + y} {2} \ qquad b = \ frac {x-y} {2} \ qquad dx \, dy = 2 \, da \, db $$

l'integrale diventa

$$ R_ {n, n} = \ frac {1} {(2 \ pi) ^ 2} \ int_0 ^ \ pi da \ int _ {- \ pi} ^ \ pi db \, \ frac {1 - \ cos (2na)} {1 - \ cos (a) \ cos (b)} $$

L'integrale $ b $ può essere fatto con la sostituzione mezza abbronzatura

$$ t = \ tan (b / 2) \ qquad \ cos (b) = \ frac {1-t ^ 2} {1 + t ^ 2} \ qquad db = \ frac {2} {1+ t ^ 2} dt $$

dare

$$ R_ {n, n} = \ frac {1} {2 \ pi} \ int_0 ^ \ pi da \, \ frac {1 - \ cos (2na)} {\ sin (a)} $$

L'identità trigonometrica

$$ 1 - \ cos (2na) = 2 \ sin (a) \ big (\ sin (a) + \ sin (3a) + \ dots + \ sin ((2n-1) a) \ big) $$

riduce il rimanente $ a $ integrale a

\ begin {align} R_ {n, n} & = \ frac {2} {\ pi} \ left (1 + \ frac {1} {3} + \ dots + \ frac {1} {2n-1} \ right) \ end {align}

Induzione

Sebbene l'integrazione fosse necessaria per ottenere i valori diagonali di $ R_ {m, n} $, il resto può essere determinato senza di essa. Il trucco sta nell'usare la simmetria rotazionale / riflessiva,

$$ R_ {n, m} = R _ {\ pm n, \ pm m} = R _ {\ pm m, \ pm n} $$

insieme alla seguente relazione di ricorrenza

$$ R_ {n + 1, m} + R_ {n-1, m} + R_ {n, m + 1} + R_ {n, m-1} - 4 R_ {n, m} = 2 \ delta _ {(n, m)} $$

che può essere dedotto utilizzando $ R_ \ vec {n} = 2 G_ \ vec {n} $ e l'equazione di definizione per $ G_ \ vec {n} $.

Inizia con la banale affermazione che

$$ R_ {0,0} = 0 $$

Applicando la relazione di ricorrenza a $ (0,0) $ e utilizzando la simmetria si ottiene

$$ R_ {1,0} = R_ {0,1} = 1/2 $$

La riga successiva è fatta in questo modo

Fill in R11, then R02 and R20

E quello successivo ...

Fill in R12 and R21, then R03 and R30

Alternando ripetutamente i due passaggi precedenti si ottiene un algoritmo per determinare ogni $ R_ {m, n} $. Chiaramente, sono tutti nella forma

$$ a + b / \ pi $$

dove $ a $ e $ b $ sono numeri razionali. Ora questo algoritmo può essere facilmente eseguito a mano, ma si potrebbe anche codificarlo in Python:

  importa numpy come np
importa frazioni come fr

N = 4
arr = np.empty ((N * 2 + 1, N * 2 + 1, 2), dtype = 'oggetto')

def plus (i, j):
    arr [i + 1, j] = 4 * arr [i, j] - arr [i - 1, j] - arr [i, j + 1] - arr [i, abs (j - 1)]

def even (i):
    arr [i, i] = arr [i - 1, i - 1] + [0, fr.fraction (2, 2 * i - 1)]
    per k nell'intervallo (1, i + 1): più (i + k - 1, i - k)

def dispari (i):
    arr [i + 1, i] = 2 * arr [i, i] - arr [i, i - 1]
    per k nell'intervallo (1, i + 1): più (i + k, i - k)

arr [0, 0] = 0
arr [1, 0] = [fr.Fraction (1, 2), 0]

per i nell'intervallo (1, N):
    perfino io)
    dispari (i)

pari (N)

per i nell'intervallo (0, N + 1):
    per j nell'intervallo (0, N + 1):
        a, b = arr [max (i, j), min (i, j)]
        print ('(', a, ') + (', b, ') / π', sep = '', end = '\ t')
    Stampa()
 

Questo produce l'output

$$ \Grande \ begin {array} {| c: c: c: c: c} 40 - \ frac {368} {3 \ pi} & \ frac {80} {\ pi} - \ frac {49} {2} & 6 - \ frac {236} {15 \ pi} & \ frac {24} {5 \ pi} - \ frac {1} {2} & \ frac {352} {105 \ pi} \\ \ hdashline \ frac {17} {2} - \ frac {24} {\ pi} & \ frac {46} {3 \ pi} - 4 & \ frac {1} {2} + \ frac {4} {3 \ pi } & \ frac {46} {15 \ pi} & \ frac {24} {5 \ pi} - \ frac {1} {2} \\ \ hdashline 2 - \ frac {4} {\ pi} & \ frac {4} {\ pi} - \ frac {1} {2} & \ frac {8} {3 \ pi} & \ frac {1} {2}+ \ frac {4} {3 \ pi} & 6 - \ frac {236} {15 \ pi} \\ \ hdashline \ frac {1} {2} & \ frac {2} {\ pi} & \ frac {4} {\ pi} - \ frac {1} {2} & \ frac {46} {3 \ pi} - 4& \ frac {80} {\ pi} - \ frac {49} {2} \\ \ hdashline 0 & \ frac {1} {2} & 2 - \ frac {4} {\ pi} & \ frac {17} {2} - \ frac {24} {\ pi} & 40 - \ frac {368} {3 \ pi} \\ \ hline \ end {array} $$

da cui possiamo leggere la risposta finale,

$$ R_ {2,1} = \ frac {4} {\ pi} - \ frac {1} {2} $$

È molto interessante che questo stesso formalismo possa essere utilizzato per risolvere la funzione a due punti di un campo scalare senza massa su un reticolo
hai dimenticato le unità


Questa domanda e risposta è stata tradotta automaticamente dalla lingua inglese. Il contenuto originale è disponibile su stackexchange, che ringraziamo per la licenza cc by-sa 2.0 con cui è distribuito.
Loading...